LSAT and Law School Admissions Forum

Get expert LSAT preparation and law school admissions advice from PowerScore Test Preparation.

User avatar
 Dave Killoran
PowerScore Staff
  • PowerScore Staff
  • Posts: 5853
  • Joined: Mar 25, 2011
|
#45766
Complete Question Explanation
(The complete setup for this game can be found here: lsat/viewtopic.php?t=8976)

The correct answer choice is (B)

The most obvious rule to check first, O in week 3, does not eliminate any of the five answer choices. Either the third or fourth rule should be used next because they are the easiest to apply visually. Answer choice (C) can be eliminated because G is not advertised with J or O, and answer choice (D) can be eliminated because K is not advertised during week 1 or 2. Next, apply the first rule, because the application of the first rule requires less work visually than the second rule. Answer choice (A) can be eliminated since J cannot be advertised during week 1, and answer choice (E) can be eliminated because J is advertised during week 2 but H is not advertised during week 1. Thus, answer choice (B) is proven correct by process of elimination. Note also that by applying the rules “out of order,” you save time because it is not necessary to apply the second rule, and the second rule would probably have required more processing time than the other rules because it forces variables to be counted.

Get the most out of your LSAT Prep Plus subscription.

Analyze and track your performance with our Testing and Analytics Package.